Question on Egoroff-like theorem












4












$begingroup$


Hi all I was tackled by this question from Folland's real analysis second edition in the second chapter, it looks like a modified Egoroff theorem but I cannot really tackle it, it is question 41 of chapter 2 which reads as follows:




Let $mu$ be a $sigma$-finite measure and $ f_n rightarrow f $ a.e. Then there exist measurable sets $E_1,E_2,ldots subset X $ such that
$$muleft(left(bigcup_{i=1}^infty E_iright)^Cright)=0$$ and $f_n rightarrow f $ uniformly on each $ E_i. $




I think this might have something to do with Egoroff's theorem but that theorem mentions nothing about complement having measure zero, only as small as you would like, which is what confuses me. Can anyone point out a proof of this with an explanation?










share|cite|improve this question











$endgroup$








  • 3




    $begingroup$
    The ideas in my answer here (math.stackexchange.com/questions/1068043/…) should be helpful.
    $endgroup$
    – PhoemueX
    Jul 31 '15 at 10:10






  • 1




    $begingroup$
    @PhoemueX thanks tried your other answer but I still cannot understand it
    $endgroup$
    – Don John Prep
    Jul 31 '15 at 11:09






  • 1




    $begingroup$
    If you start with the stronger assumption that $mu$ is a finite measure, can you see how the assertion is obtained from Egorov's theorem?
    $endgroup$
    – Daniel Fischer
    Jul 31 '15 at 13:21






  • 2




    $begingroup$
    Let $F = bigcup E_n$. Then you have $Xsetminus F subset X setminus E_k$, and hence $mu(Xsetminus F) < 1/k$. Since that holds for all $k$, $mu(Xsetminus F) = 0$. Okay, the case of finite measure done. Now, for $sigma$-finite measure, write $X = bigcup A_m$, where each $A_m$ has finite measure. Perhaps it makes things easier to assume the $A_m$ disjoint, one can do that. So by the preceding, for each $m$, we have a sequence $E^{(m)}_n$ of subsets of $A_m$. How can you use that to get the desired sequence of subsets of $X$?
    $endgroup$
    – Daniel Fischer
    Jul 31 '15 at 13:48






  • 3




    $begingroup$
    @AbeerAbassi: When you have got it worked out, you can post your own answer to this question, with your solution.
    $endgroup$
    – Nate Eldredge
    Jul 31 '15 at 14:11
















4












$begingroup$


Hi all I was tackled by this question from Folland's real analysis second edition in the second chapter, it looks like a modified Egoroff theorem but I cannot really tackle it, it is question 41 of chapter 2 which reads as follows:




Let $mu$ be a $sigma$-finite measure and $ f_n rightarrow f $ a.e. Then there exist measurable sets $E_1,E_2,ldots subset X $ such that
$$muleft(left(bigcup_{i=1}^infty E_iright)^Cright)=0$$ and $f_n rightarrow f $ uniformly on each $ E_i. $




I think this might have something to do with Egoroff's theorem but that theorem mentions nothing about complement having measure zero, only as small as you would like, which is what confuses me. Can anyone point out a proof of this with an explanation?










share|cite|improve this question











$endgroup$








  • 3




    $begingroup$
    The ideas in my answer here (math.stackexchange.com/questions/1068043/…) should be helpful.
    $endgroup$
    – PhoemueX
    Jul 31 '15 at 10:10






  • 1




    $begingroup$
    @PhoemueX thanks tried your other answer but I still cannot understand it
    $endgroup$
    – Don John Prep
    Jul 31 '15 at 11:09






  • 1




    $begingroup$
    If you start with the stronger assumption that $mu$ is a finite measure, can you see how the assertion is obtained from Egorov's theorem?
    $endgroup$
    – Daniel Fischer
    Jul 31 '15 at 13:21






  • 2




    $begingroup$
    Let $F = bigcup E_n$. Then you have $Xsetminus F subset X setminus E_k$, and hence $mu(Xsetminus F) < 1/k$. Since that holds for all $k$, $mu(Xsetminus F) = 0$. Okay, the case of finite measure done. Now, for $sigma$-finite measure, write $X = bigcup A_m$, where each $A_m$ has finite measure. Perhaps it makes things easier to assume the $A_m$ disjoint, one can do that. So by the preceding, for each $m$, we have a sequence $E^{(m)}_n$ of subsets of $A_m$. How can you use that to get the desired sequence of subsets of $X$?
    $endgroup$
    – Daniel Fischer
    Jul 31 '15 at 13:48






  • 3




    $begingroup$
    @AbeerAbassi: When you have got it worked out, you can post your own answer to this question, with your solution.
    $endgroup$
    – Nate Eldredge
    Jul 31 '15 at 14:11














4












4








4


2



$begingroup$


Hi all I was tackled by this question from Folland's real analysis second edition in the second chapter, it looks like a modified Egoroff theorem but I cannot really tackle it, it is question 41 of chapter 2 which reads as follows:




Let $mu$ be a $sigma$-finite measure and $ f_n rightarrow f $ a.e. Then there exist measurable sets $E_1,E_2,ldots subset X $ such that
$$muleft(left(bigcup_{i=1}^infty E_iright)^Cright)=0$$ and $f_n rightarrow f $ uniformly on each $ E_i. $




I think this might have something to do with Egoroff's theorem but that theorem mentions nothing about complement having measure zero, only as small as you would like, which is what confuses me. Can anyone point out a proof of this with an explanation?










share|cite|improve this question











$endgroup$




Hi all I was tackled by this question from Folland's real analysis second edition in the second chapter, it looks like a modified Egoroff theorem but I cannot really tackle it, it is question 41 of chapter 2 which reads as follows:




Let $mu$ be a $sigma$-finite measure and $ f_n rightarrow f $ a.e. Then there exist measurable sets $E_1,E_2,ldots subset X $ such that
$$muleft(left(bigcup_{i=1}^infty E_iright)^Cright)=0$$ and $f_n rightarrow f $ uniformly on each $ E_i. $




I think this might have something to do with Egoroff's theorem but that theorem mentions nothing about complement having measure zero, only as small as you would like, which is what confuses me. Can anyone point out a proof of this with an explanation?







real-analysis measure-theory uniform-convergence






share|cite|improve this question















share|cite|improve this question













share|cite|improve this question




share|cite|improve this question








edited Jul 31 '15 at 13:25









Math1000

19.2k31745




19.2k31745










asked Jul 31 '15 at 8:41









Don John PrepDon John Prep

200111




200111








  • 3




    $begingroup$
    The ideas in my answer here (math.stackexchange.com/questions/1068043/…) should be helpful.
    $endgroup$
    – PhoemueX
    Jul 31 '15 at 10:10






  • 1




    $begingroup$
    @PhoemueX thanks tried your other answer but I still cannot understand it
    $endgroup$
    – Don John Prep
    Jul 31 '15 at 11:09






  • 1




    $begingroup$
    If you start with the stronger assumption that $mu$ is a finite measure, can you see how the assertion is obtained from Egorov's theorem?
    $endgroup$
    – Daniel Fischer
    Jul 31 '15 at 13:21






  • 2




    $begingroup$
    Let $F = bigcup E_n$. Then you have $Xsetminus F subset X setminus E_k$, and hence $mu(Xsetminus F) < 1/k$. Since that holds for all $k$, $mu(Xsetminus F) = 0$. Okay, the case of finite measure done. Now, for $sigma$-finite measure, write $X = bigcup A_m$, where each $A_m$ has finite measure. Perhaps it makes things easier to assume the $A_m$ disjoint, one can do that. So by the preceding, for each $m$, we have a sequence $E^{(m)}_n$ of subsets of $A_m$. How can you use that to get the desired sequence of subsets of $X$?
    $endgroup$
    – Daniel Fischer
    Jul 31 '15 at 13:48






  • 3




    $begingroup$
    @AbeerAbassi: When you have got it worked out, you can post your own answer to this question, with your solution.
    $endgroup$
    – Nate Eldredge
    Jul 31 '15 at 14:11














  • 3




    $begingroup$
    The ideas in my answer here (math.stackexchange.com/questions/1068043/…) should be helpful.
    $endgroup$
    – PhoemueX
    Jul 31 '15 at 10:10






  • 1




    $begingroup$
    @PhoemueX thanks tried your other answer but I still cannot understand it
    $endgroup$
    – Don John Prep
    Jul 31 '15 at 11:09






  • 1




    $begingroup$
    If you start with the stronger assumption that $mu$ is a finite measure, can you see how the assertion is obtained from Egorov's theorem?
    $endgroup$
    – Daniel Fischer
    Jul 31 '15 at 13:21






  • 2




    $begingroup$
    Let $F = bigcup E_n$. Then you have $Xsetminus F subset X setminus E_k$, and hence $mu(Xsetminus F) < 1/k$. Since that holds for all $k$, $mu(Xsetminus F) = 0$. Okay, the case of finite measure done. Now, for $sigma$-finite measure, write $X = bigcup A_m$, where each $A_m$ has finite measure. Perhaps it makes things easier to assume the $A_m$ disjoint, one can do that. So by the preceding, for each $m$, we have a sequence $E^{(m)}_n$ of subsets of $A_m$. How can you use that to get the desired sequence of subsets of $X$?
    $endgroup$
    – Daniel Fischer
    Jul 31 '15 at 13:48






  • 3




    $begingroup$
    @AbeerAbassi: When you have got it worked out, you can post your own answer to this question, with your solution.
    $endgroup$
    – Nate Eldredge
    Jul 31 '15 at 14:11








3




3




$begingroup$
The ideas in my answer here (math.stackexchange.com/questions/1068043/…) should be helpful.
$endgroup$
– PhoemueX
Jul 31 '15 at 10:10




$begingroup$
The ideas in my answer here (math.stackexchange.com/questions/1068043/…) should be helpful.
$endgroup$
– PhoemueX
Jul 31 '15 at 10:10




1




1




$begingroup$
@PhoemueX thanks tried your other answer but I still cannot understand it
$endgroup$
– Don John Prep
Jul 31 '15 at 11:09




$begingroup$
@PhoemueX thanks tried your other answer but I still cannot understand it
$endgroup$
– Don John Prep
Jul 31 '15 at 11:09




1




1




$begingroup$
If you start with the stronger assumption that $mu$ is a finite measure, can you see how the assertion is obtained from Egorov's theorem?
$endgroup$
– Daniel Fischer
Jul 31 '15 at 13:21




$begingroup$
If you start with the stronger assumption that $mu$ is a finite measure, can you see how the assertion is obtained from Egorov's theorem?
$endgroup$
– Daniel Fischer
Jul 31 '15 at 13:21




2




2




$begingroup$
Let $F = bigcup E_n$. Then you have $Xsetminus F subset X setminus E_k$, and hence $mu(Xsetminus F) < 1/k$. Since that holds for all $k$, $mu(Xsetminus F) = 0$. Okay, the case of finite measure done. Now, for $sigma$-finite measure, write $X = bigcup A_m$, where each $A_m$ has finite measure. Perhaps it makes things easier to assume the $A_m$ disjoint, one can do that. So by the preceding, for each $m$, we have a sequence $E^{(m)}_n$ of subsets of $A_m$. How can you use that to get the desired sequence of subsets of $X$?
$endgroup$
– Daniel Fischer
Jul 31 '15 at 13:48




$begingroup$
Let $F = bigcup E_n$. Then you have $Xsetminus F subset X setminus E_k$, and hence $mu(Xsetminus F) < 1/k$. Since that holds for all $k$, $mu(Xsetminus F) = 0$. Okay, the case of finite measure done. Now, for $sigma$-finite measure, write $X = bigcup A_m$, where each $A_m$ has finite measure. Perhaps it makes things easier to assume the $A_m$ disjoint, one can do that. So by the preceding, for each $m$, we have a sequence $E^{(m)}_n$ of subsets of $A_m$. How can you use that to get the desired sequence of subsets of $X$?
$endgroup$
– Daniel Fischer
Jul 31 '15 at 13:48




3




3




$begingroup$
@AbeerAbassi: When you have got it worked out, you can post your own answer to this question, with your solution.
$endgroup$
– Nate Eldredge
Jul 31 '15 at 14:11




$begingroup$
@AbeerAbassi: When you have got it worked out, you can post your own answer to this question, with your solution.
$endgroup$
– Nate Eldredge
Jul 31 '15 at 14:11










1 Answer
1






active

oldest

votes


















1












$begingroup$

Since $mu$ is a $sigma-$finite measure, there exists ${A_n}_{nin mathbb{N}}$ such that $bigcup_{n}A_n=X$ and $mu(A_n)<infty$ $forall n$.



Let $epsilon>0$ be given. Note that $f_nrightarrow f$ a.e on each $A_n$. Therefore, by Egoroff's theorem, there exists $E_nsubset A_n$ such that $mu(A_nsetminus E_n)<frac{epsilon}{2^n}$ and $f_n rightarrow f$ uniformly on $E_n$ for each $nin mathbb{N}$.



Now, observe that



begin{align*}
mu(left(bigcup_{n}E_n right)^c)&= muleft( bigcup_n A_n setminus bigcup_n E_n right)\ &= muleft(bigcup_{n}(A_nsetminus E_n)right)\&leq sum_n mu(A_nsetminus E_n)\&<sum_nfrac{epsilon}{2^n}=epsilon hspace{1 in}forall nin mathbb{N}.
end{align*}



Since $epsilon >0$ is arbitrary, $mu(left(bigcup_{n}E_n right)^c)=0$.



Since $f_nrightarrow f$ uniformly on each $E_n$ by construction, we are done.






share|cite|improve this answer











$endgroup$









  • 1




    $begingroup$
    Hi, when you write $mu(E_n^c) < frac{1}{n}$ you actually mean $mu(A_n - E_n)$. Thus your argument doesn't work as stated. But you can actually prove that Egoroff works for $sigma- $ finite case by using your construction and using a standard $frac{epsilon}{2^n}$ argument.
    $endgroup$
    – Soham
    Dec 11 '18 at 5:36










  • $begingroup$
    @LucyferZedd I did not realize that! I fixed it :) Thank you!
    $endgroup$
    – Lev Ban
    Dec 14 '18 at 16:06











Your Answer





StackExchange.ifUsing("editor", function () {
return StackExchange.using("mathjaxEditing", function () {
StackExchange.MarkdownEditor.creationCallbacks.add(function (editor, postfix) {
StackExchange.mathjaxEditing.prepareWmdForMathJax(editor, postfix, [["$", "$"], ["\\(","\\)"]]);
});
});
}, "mathjax-editing");

StackExchange.ready(function() {
var channelOptions = {
tags: "".split(" "),
id: "69"
};
initTagRenderer("".split(" "), "".split(" "), channelOptions);

StackExchange.using("externalEditor", function() {
// Have to fire editor after snippets, if snippets enabled
if (StackExchange.settings.snippets.snippetsEnabled) {
StackExchange.using("snippets", function() {
createEditor();
});
}
else {
createEditor();
}
});

function createEditor() {
StackExchange.prepareEditor({
heartbeatType: 'answer',
autoActivateHeartbeat: false,
convertImagesToLinks: true,
noModals: true,
showLowRepImageUploadWarning: true,
reputationToPostImages: 10,
bindNavPrevention: true,
postfix: "",
imageUploader: {
brandingHtml: "Powered by u003ca class="icon-imgur-white" href="https://imgur.com/"u003eu003c/au003e",
contentPolicyHtml: "User contributions licensed under u003ca href="https://creativecommons.org/licenses/by-sa/3.0/"u003ecc by-sa 3.0 with attribution requiredu003c/au003e u003ca href="https://stackoverflow.com/legal/content-policy"u003e(content policy)u003c/au003e",
allowUrls: true
},
noCode: true, onDemand: true,
discardSelector: ".discard-answer"
,immediatelyShowMarkdownHelp:true
});


}
});














draft saved

draft discarded


















StackExchange.ready(
function () {
StackExchange.openid.initPostLogin('.new-post-login', 'https%3a%2f%2fmath.stackexchange.com%2fquestions%2f1380077%2fquestion-on-egoroff-like-theorem%23new-answer', 'question_page');
}
);

Post as a guest















Required, but never shown

























1 Answer
1






active

oldest

votes








1 Answer
1






active

oldest

votes









active

oldest

votes






active

oldest

votes









1












$begingroup$

Since $mu$ is a $sigma-$finite measure, there exists ${A_n}_{nin mathbb{N}}$ such that $bigcup_{n}A_n=X$ and $mu(A_n)<infty$ $forall n$.



Let $epsilon>0$ be given. Note that $f_nrightarrow f$ a.e on each $A_n$. Therefore, by Egoroff's theorem, there exists $E_nsubset A_n$ such that $mu(A_nsetminus E_n)<frac{epsilon}{2^n}$ and $f_n rightarrow f$ uniformly on $E_n$ for each $nin mathbb{N}$.



Now, observe that



begin{align*}
mu(left(bigcup_{n}E_n right)^c)&= muleft( bigcup_n A_n setminus bigcup_n E_n right)\ &= muleft(bigcup_{n}(A_nsetminus E_n)right)\&leq sum_n mu(A_nsetminus E_n)\&<sum_nfrac{epsilon}{2^n}=epsilon hspace{1 in}forall nin mathbb{N}.
end{align*}



Since $epsilon >0$ is arbitrary, $mu(left(bigcup_{n}E_n right)^c)=0$.



Since $f_nrightarrow f$ uniformly on each $E_n$ by construction, we are done.






share|cite|improve this answer











$endgroup$









  • 1




    $begingroup$
    Hi, when you write $mu(E_n^c) < frac{1}{n}$ you actually mean $mu(A_n - E_n)$. Thus your argument doesn't work as stated. But you can actually prove that Egoroff works for $sigma- $ finite case by using your construction and using a standard $frac{epsilon}{2^n}$ argument.
    $endgroup$
    – Soham
    Dec 11 '18 at 5:36










  • $begingroup$
    @LucyferZedd I did not realize that! I fixed it :) Thank you!
    $endgroup$
    – Lev Ban
    Dec 14 '18 at 16:06
















1












$begingroup$

Since $mu$ is a $sigma-$finite measure, there exists ${A_n}_{nin mathbb{N}}$ such that $bigcup_{n}A_n=X$ and $mu(A_n)<infty$ $forall n$.



Let $epsilon>0$ be given. Note that $f_nrightarrow f$ a.e on each $A_n$. Therefore, by Egoroff's theorem, there exists $E_nsubset A_n$ such that $mu(A_nsetminus E_n)<frac{epsilon}{2^n}$ and $f_n rightarrow f$ uniformly on $E_n$ for each $nin mathbb{N}$.



Now, observe that



begin{align*}
mu(left(bigcup_{n}E_n right)^c)&= muleft( bigcup_n A_n setminus bigcup_n E_n right)\ &= muleft(bigcup_{n}(A_nsetminus E_n)right)\&leq sum_n mu(A_nsetminus E_n)\&<sum_nfrac{epsilon}{2^n}=epsilon hspace{1 in}forall nin mathbb{N}.
end{align*}



Since $epsilon >0$ is arbitrary, $mu(left(bigcup_{n}E_n right)^c)=0$.



Since $f_nrightarrow f$ uniformly on each $E_n$ by construction, we are done.






share|cite|improve this answer











$endgroup$









  • 1




    $begingroup$
    Hi, when you write $mu(E_n^c) < frac{1}{n}$ you actually mean $mu(A_n - E_n)$. Thus your argument doesn't work as stated. But you can actually prove that Egoroff works for $sigma- $ finite case by using your construction and using a standard $frac{epsilon}{2^n}$ argument.
    $endgroup$
    – Soham
    Dec 11 '18 at 5:36










  • $begingroup$
    @LucyferZedd I did not realize that! I fixed it :) Thank you!
    $endgroup$
    – Lev Ban
    Dec 14 '18 at 16:06














1












1








1





$begingroup$

Since $mu$ is a $sigma-$finite measure, there exists ${A_n}_{nin mathbb{N}}$ such that $bigcup_{n}A_n=X$ and $mu(A_n)<infty$ $forall n$.



Let $epsilon>0$ be given. Note that $f_nrightarrow f$ a.e on each $A_n$. Therefore, by Egoroff's theorem, there exists $E_nsubset A_n$ such that $mu(A_nsetminus E_n)<frac{epsilon}{2^n}$ and $f_n rightarrow f$ uniformly on $E_n$ for each $nin mathbb{N}$.



Now, observe that



begin{align*}
mu(left(bigcup_{n}E_n right)^c)&= muleft( bigcup_n A_n setminus bigcup_n E_n right)\ &= muleft(bigcup_{n}(A_nsetminus E_n)right)\&leq sum_n mu(A_nsetminus E_n)\&<sum_nfrac{epsilon}{2^n}=epsilon hspace{1 in}forall nin mathbb{N}.
end{align*}



Since $epsilon >0$ is arbitrary, $mu(left(bigcup_{n}E_n right)^c)=0$.



Since $f_nrightarrow f$ uniformly on each $E_n$ by construction, we are done.






share|cite|improve this answer











$endgroup$



Since $mu$ is a $sigma-$finite measure, there exists ${A_n}_{nin mathbb{N}}$ such that $bigcup_{n}A_n=X$ and $mu(A_n)<infty$ $forall n$.



Let $epsilon>0$ be given. Note that $f_nrightarrow f$ a.e on each $A_n$. Therefore, by Egoroff's theorem, there exists $E_nsubset A_n$ such that $mu(A_nsetminus E_n)<frac{epsilon}{2^n}$ and $f_n rightarrow f$ uniformly on $E_n$ for each $nin mathbb{N}$.



Now, observe that



begin{align*}
mu(left(bigcup_{n}E_n right)^c)&= muleft( bigcup_n A_n setminus bigcup_n E_n right)\ &= muleft(bigcup_{n}(A_nsetminus E_n)right)\&leq sum_n mu(A_nsetminus E_n)\&<sum_nfrac{epsilon}{2^n}=epsilon hspace{1 in}forall nin mathbb{N}.
end{align*}



Since $epsilon >0$ is arbitrary, $mu(left(bigcup_{n}E_n right)^c)=0$.



Since $f_nrightarrow f$ uniformly on each $E_n$ by construction, we are done.







share|cite|improve this answer














share|cite|improve this answer



share|cite|improve this answer








edited Dec 14 '18 at 16:05

























answered Aug 6 '18 at 14:31









Lev BanLev Ban

1,0671317




1,0671317








  • 1




    $begingroup$
    Hi, when you write $mu(E_n^c) < frac{1}{n}$ you actually mean $mu(A_n - E_n)$. Thus your argument doesn't work as stated. But you can actually prove that Egoroff works for $sigma- $ finite case by using your construction and using a standard $frac{epsilon}{2^n}$ argument.
    $endgroup$
    – Soham
    Dec 11 '18 at 5:36










  • $begingroup$
    @LucyferZedd I did not realize that! I fixed it :) Thank you!
    $endgroup$
    – Lev Ban
    Dec 14 '18 at 16:06














  • 1




    $begingroup$
    Hi, when you write $mu(E_n^c) < frac{1}{n}$ you actually mean $mu(A_n - E_n)$. Thus your argument doesn't work as stated. But you can actually prove that Egoroff works for $sigma- $ finite case by using your construction and using a standard $frac{epsilon}{2^n}$ argument.
    $endgroup$
    – Soham
    Dec 11 '18 at 5:36










  • $begingroup$
    @LucyferZedd I did not realize that! I fixed it :) Thank you!
    $endgroup$
    – Lev Ban
    Dec 14 '18 at 16:06








1




1




$begingroup$
Hi, when you write $mu(E_n^c) < frac{1}{n}$ you actually mean $mu(A_n - E_n)$. Thus your argument doesn't work as stated. But you can actually prove that Egoroff works for $sigma- $ finite case by using your construction and using a standard $frac{epsilon}{2^n}$ argument.
$endgroup$
– Soham
Dec 11 '18 at 5:36




$begingroup$
Hi, when you write $mu(E_n^c) < frac{1}{n}$ you actually mean $mu(A_n - E_n)$. Thus your argument doesn't work as stated. But you can actually prove that Egoroff works for $sigma- $ finite case by using your construction and using a standard $frac{epsilon}{2^n}$ argument.
$endgroup$
– Soham
Dec 11 '18 at 5:36












$begingroup$
@LucyferZedd I did not realize that! I fixed it :) Thank you!
$endgroup$
– Lev Ban
Dec 14 '18 at 16:06




$begingroup$
@LucyferZedd I did not realize that! I fixed it :) Thank you!
$endgroup$
– Lev Ban
Dec 14 '18 at 16:06


















draft saved

draft discarded




















































Thanks for contributing an answer to Mathematics Stack Exchange!


  • Please be sure to answer the question. Provide details and share your research!

But avoid



  • Asking for help, clarification, or responding to other answers.

  • Making statements based on opinion; back them up with references or personal experience.


Use MathJax to format equations. MathJax reference.


To learn more, see our tips on writing great answers.




draft saved


draft discarded














StackExchange.ready(
function () {
StackExchange.openid.initPostLogin('.new-post-login', 'https%3a%2f%2fmath.stackexchange.com%2fquestions%2f1380077%2fquestion-on-egoroff-like-theorem%23new-answer', 'question_page');
}
);

Post as a guest















Required, but never shown





















































Required, but never shown














Required, but never shown












Required, but never shown







Required, but never shown

































Required, but never shown














Required, but never shown












Required, but never shown







Required, but never shown







Popular posts from this blog

Le Mesnil-Réaume

Ida-Boy-Ed-Garten

web3.py web3.isConnected() returns false always